Cartesian product of Lebesgue measurable sets is Lebesgue measurable












0












$begingroup$


Let $A subset mathbb{R^m}$, $B subset mathbb{R^n}$ and $lambda_n$ the Lebesgue measure on $mathbb{R^n}$.



How to prove that if $A$ and $B$ are Lebesgue measurable sets then also $A times B$ is Lebesgue measurable.



For showing that the cartesian product is Lebesgue measurable, I tried to use:



$A times B$ is Lebesgue measurable if $varepsilon>0$, and an open set $U$ and a closed set $V$ exist such that $V subset A times B subset U$ and $mathcal{L^{n+m}}(U$ $V)<varepsilon$.



I don't know how to continue from here and how to conclude that the cartesian product is also Lebesgue measurable.










share|cite|improve this question











$endgroup$












  • $begingroup$
    You are probably discussing measurable with respect to $mathcal{L}{n+m}$ measure on $mathbb{R}^{n+m}$, or am I wrong?
    $endgroup$
    – Keen-ameteur
    Nov 26 '18 at 17:27










  • $begingroup$
    Let $A subset mathbb{R^m}$, $B subset mathbb{R^n}$ and $lambda_n$ , thats all what is given, so I suppose it could be right.
    $endgroup$
    – Olsgur
    Nov 26 '18 at 18:54










  • $begingroup$
    What does $mathcal{L}^{n+m}(Usetminus V)< epsilon$ then? It must be the Lebesgue measure on $mathbb{R}^{n+m}$.
    $endgroup$
    – Keen-ameteur
    Nov 26 '18 at 19:15








  • 1




    $begingroup$
    By the way are you familiar with the terms $sigma$ algebra and product $sigma$ algebra?
    $endgroup$
    – Keen-ameteur
    Nov 26 '18 at 19:22










  • $begingroup$
    I know the definitions, but I haven't really worked with them yet, especially not with the product sigma algebra.
    $endgroup$
    – Olsgur
    Nov 26 '18 at 19:54


















0












$begingroup$


Let $A subset mathbb{R^m}$, $B subset mathbb{R^n}$ and $lambda_n$ the Lebesgue measure on $mathbb{R^n}$.



How to prove that if $A$ and $B$ are Lebesgue measurable sets then also $A times B$ is Lebesgue measurable.



For showing that the cartesian product is Lebesgue measurable, I tried to use:



$A times B$ is Lebesgue measurable if $varepsilon>0$, and an open set $U$ and a closed set $V$ exist such that $V subset A times B subset U$ and $mathcal{L^{n+m}}(U$ $V)<varepsilon$.



I don't know how to continue from here and how to conclude that the cartesian product is also Lebesgue measurable.










share|cite|improve this question











$endgroup$












  • $begingroup$
    You are probably discussing measurable with respect to $mathcal{L}{n+m}$ measure on $mathbb{R}^{n+m}$, or am I wrong?
    $endgroup$
    – Keen-ameteur
    Nov 26 '18 at 17:27










  • $begingroup$
    Let $A subset mathbb{R^m}$, $B subset mathbb{R^n}$ and $lambda_n$ , thats all what is given, so I suppose it could be right.
    $endgroup$
    – Olsgur
    Nov 26 '18 at 18:54










  • $begingroup$
    What does $mathcal{L}^{n+m}(Usetminus V)< epsilon$ then? It must be the Lebesgue measure on $mathbb{R}^{n+m}$.
    $endgroup$
    – Keen-ameteur
    Nov 26 '18 at 19:15








  • 1




    $begingroup$
    By the way are you familiar with the terms $sigma$ algebra and product $sigma$ algebra?
    $endgroup$
    – Keen-ameteur
    Nov 26 '18 at 19:22










  • $begingroup$
    I know the definitions, but I haven't really worked with them yet, especially not with the product sigma algebra.
    $endgroup$
    – Olsgur
    Nov 26 '18 at 19:54
















0












0








0





$begingroup$


Let $A subset mathbb{R^m}$, $B subset mathbb{R^n}$ and $lambda_n$ the Lebesgue measure on $mathbb{R^n}$.



How to prove that if $A$ and $B$ are Lebesgue measurable sets then also $A times B$ is Lebesgue measurable.



For showing that the cartesian product is Lebesgue measurable, I tried to use:



$A times B$ is Lebesgue measurable if $varepsilon>0$, and an open set $U$ and a closed set $V$ exist such that $V subset A times B subset U$ and $mathcal{L^{n+m}}(U$ $V)<varepsilon$.



I don't know how to continue from here and how to conclude that the cartesian product is also Lebesgue measurable.










share|cite|improve this question











$endgroup$




Let $A subset mathbb{R^m}$, $B subset mathbb{R^n}$ and $lambda_n$ the Lebesgue measure on $mathbb{R^n}$.



How to prove that if $A$ and $B$ are Lebesgue measurable sets then also $A times B$ is Lebesgue measurable.



For showing that the cartesian product is Lebesgue measurable, I tried to use:



$A times B$ is Lebesgue measurable if $varepsilon>0$, and an open set $U$ and a closed set $V$ exist such that $V subset A times B subset U$ and $mathcal{L^{n+m}}(U$ $V)<varepsilon$.



I don't know how to continue from here and how to conclude that the cartesian product is also Lebesgue measurable.







measure-theory lebesgue-measure






share|cite|improve this question















share|cite|improve this question













share|cite|improve this question




share|cite|improve this question








edited Nov 26 '18 at 18:56







Olsgur

















asked Nov 26 '18 at 17:13









OlsgurOlsgur

544




544












  • $begingroup$
    You are probably discussing measurable with respect to $mathcal{L}{n+m}$ measure on $mathbb{R}^{n+m}$, or am I wrong?
    $endgroup$
    – Keen-ameteur
    Nov 26 '18 at 17:27










  • $begingroup$
    Let $A subset mathbb{R^m}$, $B subset mathbb{R^n}$ and $lambda_n$ , thats all what is given, so I suppose it could be right.
    $endgroup$
    – Olsgur
    Nov 26 '18 at 18:54










  • $begingroup$
    What does $mathcal{L}^{n+m}(Usetminus V)< epsilon$ then? It must be the Lebesgue measure on $mathbb{R}^{n+m}$.
    $endgroup$
    – Keen-ameteur
    Nov 26 '18 at 19:15








  • 1




    $begingroup$
    By the way are you familiar with the terms $sigma$ algebra and product $sigma$ algebra?
    $endgroup$
    – Keen-ameteur
    Nov 26 '18 at 19:22










  • $begingroup$
    I know the definitions, but I haven't really worked with them yet, especially not with the product sigma algebra.
    $endgroup$
    – Olsgur
    Nov 26 '18 at 19:54




















  • $begingroup$
    You are probably discussing measurable with respect to $mathcal{L}{n+m}$ measure on $mathbb{R}^{n+m}$, or am I wrong?
    $endgroup$
    – Keen-ameteur
    Nov 26 '18 at 17:27










  • $begingroup$
    Let $A subset mathbb{R^m}$, $B subset mathbb{R^n}$ and $lambda_n$ , thats all what is given, so I suppose it could be right.
    $endgroup$
    – Olsgur
    Nov 26 '18 at 18:54










  • $begingroup$
    What does $mathcal{L}^{n+m}(Usetminus V)< epsilon$ then? It must be the Lebesgue measure on $mathbb{R}^{n+m}$.
    $endgroup$
    – Keen-ameteur
    Nov 26 '18 at 19:15








  • 1




    $begingroup$
    By the way are you familiar with the terms $sigma$ algebra and product $sigma$ algebra?
    $endgroup$
    – Keen-ameteur
    Nov 26 '18 at 19:22










  • $begingroup$
    I know the definitions, but I haven't really worked with them yet, especially not with the product sigma algebra.
    $endgroup$
    – Olsgur
    Nov 26 '18 at 19:54


















$begingroup$
You are probably discussing measurable with respect to $mathcal{L}{n+m}$ measure on $mathbb{R}^{n+m}$, or am I wrong?
$endgroup$
– Keen-ameteur
Nov 26 '18 at 17:27




$begingroup$
You are probably discussing measurable with respect to $mathcal{L}{n+m}$ measure on $mathbb{R}^{n+m}$, or am I wrong?
$endgroup$
– Keen-ameteur
Nov 26 '18 at 17:27












$begingroup$
Let $A subset mathbb{R^m}$, $B subset mathbb{R^n}$ and $lambda_n$ , thats all what is given, so I suppose it could be right.
$endgroup$
– Olsgur
Nov 26 '18 at 18:54




$begingroup$
Let $A subset mathbb{R^m}$, $B subset mathbb{R^n}$ and $lambda_n$ , thats all what is given, so I suppose it could be right.
$endgroup$
– Olsgur
Nov 26 '18 at 18:54












$begingroup$
What does $mathcal{L}^{n+m}(Usetminus V)< epsilon$ then? It must be the Lebesgue measure on $mathbb{R}^{n+m}$.
$endgroup$
– Keen-ameteur
Nov 26 '18 at 19:15






$begingroup$
What does $mathcal{L}^{n+m}(Usetminus V)< epsilon$ then? It must be the Lebesgue measure on $mathbb{R}^{n+m}$.
$endgroup$
– Keen-ameteur
Nov 26 '18 at 19:15






1




1




$begingroup$
By the way are you familiar with the terms $sigma$ algebra and product $sigma$ algebra?
$endgroup$
– Keen-ameteur
Nov 26 '18 at 19:22




$begingroup$
By the way are you familiar with the terms $sigma$ algebra and product $sigma$ algebra?
$endgroup$
– Keen-ameteur
Nov 26 '18 at 19:22












$begingroup$
I know the definitions, but I haven't really worked with them yet, especially not with the product sigma algebra.
$endgroup$
– Olsgur
Nov 26 '18 at 19:54






$begingroup$
I know the definitions, but I haven't really worked with them yet, especially not with the product sigma algebra.
$endgroup$
– Olsgur
Nov 26 '18 at 19:54












1 Answer
1






active

oldest

votes


















0












$begingroup$

As a prelude, let me first say that $Atimes B$ is clearly in the product $sigma$ algebra, since the product sigma algebra $mathcal{F}otimes mathcal{G}$ of two measurable spaces is the sigma algebra generated by measurable 'rectangles'. i.e:



$mathcal{F}otimes mathcal{G}= sigmaBig( { Etimes F: Ein mathcal{F}, Fin mathcal{G} } Big)$



Going by this definition, your set $Atimes B$ is Lebesgue measurable in $mathbb{R}^{n+m}$. However going by your criterion, recall first that for $Ain mathcal{L}^n$ and $Bin mathcal{L}^m$, we have:



$lambda_n otimes lambda_m (Atimes B)= lambda_n(A)times lambda_m(B)$



Since $A,B$ are Lebesgue measurable, there exist $U_1,U_2$ open and $V_1,V_2$ closed such that:



$U_1supseteq A supseteq V_1$ , $U_2supseteq B supseteq V_2$ while $lambda_n(U_1setminus V_1)leq frac{1}{2} tilde{epsilon}$ and $lambda_m(U_2setminus V_2)leq frac{1}{2} tilde{epsilon}$



Notice that $U_1times U_2supseteq Atimes B supseteq V_1times V_2$ while $U_1times U_2$ is open and $V_1 times V_2$ is closed. Since you can write:



$U_1times U_2= Big( U_1times V_2 Big) sqcup Big( U_1 times (U_2 setminus V_2) Big)= Big( V_1times V_2 Big) sqcup Big( (U_1setminus V_1)times V_2 Big) sqcup Big( U_1 times (U_2setminus V_2) Big) $



Then:



$lambda_{n+m}Big( (U_1times U_2) setminus (V_1 times V_2) Big)= lambda_{n+m}Big( (U_1setminus V_1)times V_2
Big) + lambda_{n+m}Big( U_1 times (U_2setminus V_2)
Big)= $



$= lambda_n(U_1 setminus V_1)cdot lambda_m(V_2)+ lambda_n(U)cdot lambda_m(U_2 setminus V_2)$



If you assume $lambda_n(U_1), lambda_m(U_2)leq M$, then for $tilde{epsilon}= dfrac{epsilon}{2M}$, you've shown that the difference is of measure less than $epsilon$. Otherwise, there is a standard argument (which I will elaborate if you request) of going from the finite measured case to the $sigma$-finite case.






share|cite|improve this answer









$endgroup$













  • $begingroup$
    Thanks for the explanation. If there is a standard way to show it, how would it look like in this case?
    $endgroup$
    – Olsgur
    Nov 27 '18 at 13:56










  • $begingroup$
    I'm unclear on what exactly do you mean by showing in a standard way? The 'standard argument'?
    $endgroup$
    – Keen-ameteur
    Nov 27 '18 at 18:34










  • $begingroup$
    Yes, I mean the standard argument with the $sigma$-finite case.
    $endgroup$
    – Olsgur
    Nov 27 '18 at 23:23











Your Answer





StackExchange.ifUsing("editor", function () {
return StackExchange.using("mathjaxEditing", function () {
StackExchange.MarkdownEditor.creationCallbacks.add(function (editor, postfix) {
StackExchange.mathjaxEditing.prepareWmdForMathJax(editor, postfix, [["$", "$"], ["\\(","\\)"]]);
});
});
}, "mathjax-editing");

StackExchange.ready(function() {
var channelOptions = {
tags: "".split(" "),
id: "69"
};
initTagRenderer("".split(" "), "".split(" "), channelOptions);

StackExchange.using("externalEditor", function() {
// Have to fire editor after snippets, if snippets enabled
if (StackExchange.settings.snippets.snippetsEnabled) {
StackExchange.using("snippets", function() {
createEditor();
});
}
else {
createEditor();
}
});

function createEditor() {
StackExchange.prepareEditor({
heartbeatType: 'answer',
autoActivateHeartbeat: false,
convertImagesToLinks: true,
noModals: true,
showLowRepImageUploadWarning: true,
reputationToPostImages: 10,
bindNavPrevention: true,
postfix: "",
imageUploader: {
brandingHtml: "Powered by u003ca class="icon-imgur-white" href="https://imgur.com/"u003eu003c/au003e",
contentPolicyHtml: "User contributions licensed under u003ca href="https://creativecommons.org/licenses/by-sa/3.0/"u003ecc by-sa 3.0 with attribution requiredu003c/au003e u003ca href="https://stackoverflow.com/legal/content-policy"u003e(content policy)u003c/au003e",
allowUrls: true
},
noCode: true, onDemand: true,
discardSelector: ".discard-answer"
,immediatelyShowMarkdownHelp:true
});


}
});














draft saved

draft discarded


















StackExchange.ready(
function () {
StackExchange.openid.initPostLogin('.new-post-login', 'https%3a%2f%2fmath.stackexchange.com%2fquestions%2f3014610%2fcartesian-product-of-lebesgue-measurable-sets-is-lebesgue-measurable%23new-answer', 'question_page');
}
);

Post as a guest















Required, but never shown

























1 Answer
1






active

oldest

votes








1 Answer
1






active

oldest

votes









active

oldest

votes






active

oldest

votes









0












$begingroup$

As a prelude, let me first say that $Atimes B$ is clearly in the product $sigma$ algebra, since the product sigma algebra $mathcal{F}otimes mathcal{G}$ of two measurable spaces is the sigma algebra generated by measurable 'rectangles'. i.e:



$mathcal{F}otimes mathcal{G}= sigmaBig( { Etimes F: Ein mathcal{F}, Fin mathcal{G} } Big)$



Going by this definition, your set $Atimes B$ is Lebesgue measurable in $mathbb{R}^{n+m}$. However going by your criterion, recall first that for $Ain mathcal{L}^n$ and $Bin mathcal{L}^m$, we have:



$lambda_n otimes lambda_m (Atimes B)= lambda_n(A)times lambda_m(B)$



Since $A,B$ are Lebesgue measurable, there exist $U_1,U_2$ open and $V_1,V_2$ closed such that:



$U_1supseteq A supseteq V_1$ , $U_2supseteq B supseteq V_2$ while $lambda_n(U_1setminus V_1)leq frac{1}{2} tilde{epsilon}$ and $lambda_m(U_2setminus V_2)leq frac{1}{2} tilde{epsilon}$



Notice that $U_1times U_2supseteq Atimes B supseteq V_1times V_2$ while $U_1times U_2$ is open and $V_1 times V_2$ is closed. Since you can write:



$U_1times U_2= Big( U_1times V_2 Big) sqcup Big( U_1 times (U_2 setminus V_2) Big)= Big( V_1times V_2 Big) sqcup Big( (U_1setminus V_1)times V_2 Big) sqcup Big( U_1 times (U_2setminus V_2) Big) $



Then:



$lambda_{n+m}Big( (U_1times U_2) setminus (V_1 times V_2) Big)= lambda_{n+m}Big( (U_1setminus V_1)times V_2
Big) + lambda_{n+m}Big( U_1 times (U_2setminus V_2)
Big)= $



$= lambda_n(U_1 setminus V_1)cdot lambda_m(V_2)+ lambda_n(U)cdot lambda_m(U_2 setminus V_2)$



If you assume $lambda_n(U_1), lambda_m(U_2)leq M$, then for $tilde{epsilon}= dfrac{epsilon}{2M}$, you've shown that the difference is of measure less than $epsilon$. Otherwise, there is a standard argument (which I will elaborate if you request) of going from the finite measured case to the $sigma$-finite case.






share|cite|improve this answer









$endgroup$













  • $begingroup$
    Thanks for the explanation. If there is a standard way to show it, how would it look like in this case?
    $endgroup$
    – Olsgur
    Nov 27 '18 at 13:56










  • $begingroup$
    I'm unclear on what exactly do you mean by showing in a standard way? The 'standard argument'?
    $endgroup$
    – Keen-ameteur
    Nov 27 '18 at 18:34










  • $begingroup$
    Yes, I mean the standard argument with the $sigma$-finite case.
    $endgroup$
    – Olsgur
    Nov 27 '18 at 23:23
















0












$begingroup$

As a prelude, let me first say that $Atimes B$ is clearly in the product $sigma$ algebra, since the product sigma algebra $mathcal{F}otimes mathcal{G}$ of two measurable spaces is the sigma algebra generated by measurable 'rectangles'. i.e:



$mathcal{F}otimes mathcal{G}= sigmaBig( { Etimes F: Ein mathcal{F}, Fin mathcal{G} } Big)$



Going by this definition, your set $Atimes B$ is Lebesgue measurable in $mathbb{R}^{n+m}$. However going by your criterion, recall first that for $Ain mathcal{L}^n$ and $Bin mathcal{L}^m$, we have:



$lambda_n otimes lambda_m (Atimes B)= lambda_n(A)times lambda_m(B)$



Since $A,B$ are Lebesgue measurable, there exist $U_1,U_2$ open and $V_1,V_2$ closed such that:



$U_1supseteq A supseteq V_1$ , $U_2supseteq B supseteq V_2$ while $lambda_n(U_1setminus V_1)leq frac{1}{2} tilde{epsilon}$ and $lambda_m(U_2setminus V_2)leq frac{1}{2} tilde{epsilon}$



Notice that $U_1times U_2supseteq Atimes B supseteq V_1times V_2$ while $U_1times U_2$ is open and $V_1 times V_2$ is closed. Since you can write:



$U_1times U_2= Big( U_1times V_2 Big) sqcup Big( U_1 times (U_2 setminus V_2) Big)= Big( V_1times V_2 Big) sqcup Big( (U_1setminus V_1)times V_2 Big) sqcup Big( U_1 times (U_2setminus V_2) Big) $



Then:



$lambda_{n+m}Big( (U_1times U_2) setminus (V_1 times V_2) Big)= lambda_{n+m}Big( (U_1setminus V_1)times V_2
Big) + lambda_{n+m}Big( U_1 times (U_2setminus V_2)
Big)= $



$= lambda_n(U_1 setminus V_1)cdot lambda_m(V_2)+ lambda_n(U)cdot lambda_m(U_2 setminus V_2)$



If you assume $lambda_n(U_1), lambda_m(U_2)leq M$, then for $tilde{epsilon}= dfrac{epsilon}{2M}$, you've shown that the difference is of measure less than $epsilon$. Otherwise, there is a standard argument (which I will elaborate if you request) of going from the finite measured case to the $sigma$-finite case.






share|cite|improve this answer









$endgroup$













  • $begingroup$
    Thanks for the explanation. If there is a standard way to show it, how would it look like in this case?
    $endgroup$
    – Olsgur
    Nov 27 '18 at 13:56










  • $begingroup$
    I'm unclear on what exactly do you mean by showing in a standard way? The 'standard argument'?
    $endgroup$
    – Keen-ameteur
    Nov 27 '18 at 18:34










  • $begingroup$
    Yes, I mean the standard argument with the $sigma$-finite case.
    $endgroup$
    – Olsgur
    Nov 27 '18 at 23:23














0












0








0





$begingroup$

As a prelude, let me first say that $Atimes B$ is clearly in the product $sigma$ algebra, since the product sigma algebra $mathcal{F}otimes mathcal{G}$ of two measurable spaces is the sigma algebra generated by measurable 'rectangles'. i.e:



$mathcal{F}otimes mathcal{G}= sigmaBig( { Etimes F: Ein mathcal{F}, Fin mathcal{G} } Big)$



Going by this definition, your set $Atimes B$ is Lebesgue measurable in $mathbb{R}^{n+m}$. However going by your criterion, recall first that for $Ain mathcal{L}^n$ and $Bin mathcal{L}^m$, we have:



$lambda_n otimes lambda_m (Atimes B)= lambda_n(A)times lambda_m(B)$



Since $A,B$ are Lebesgue measurable, there exist $U_1,U_2$ open and $V_1,V_2$ closed such that:



$U_1supseteq A supseteq V_1$ , $U_2supseteq B supseteq V_2$ while $lambda_n(U_1setminus V_1)leq frac{1}{2} tilde{epsilon}$ and $lambda_m(U_2setminus V_2)leq frac{1}{2} tilde{epsilon}$



Notice that $U_1times U_2supseteq Atimes B supseteq V_1times V_2$ while $U_1times U_2$ is open and $V_1 times V_2$ is closed. Since you can write:



$U_1times U_2= Big( U_1times V_2 Big) sqcup Big( U_1 times (U_2 setminus V_2) Big)= Big( V_1times V_2 Big) sqcup Big( (U_1setminus V_1)times V_2 Big) sqcup Big( U_1 times (U_2setminus V_2) Big) $



Then:



$lambda_{n+m}Big( (U_1times U_2) setminus (V_1 times V_2) Big)= lambda_{n+m}Big( (U_1setminus V_1)times V_2
Big) + lambda_{n+m}Big( U_1 times (U_2setminus V_2)
Big)= $



$= lambda_n(U_1 setminus V_1)cdot lambda_m(V_2)+ lambda_n(U)cdot lambda_m(U_2 setminus V_2)$



If you assume $lambda_n(U_1), lambda_m(U_2)leq M$, then for $tilde{epsilon}= dfrac{epsilon}{2M}$, you've shown that the difference is of measure less than $epsilon$. Otherwise, there is a standard argument (which I will elaborate if you request) of going from the finite measured case to the $sigma$-finite case.






share|cite|improve this answer









$endgroup$



As a prelude, let me first say that $Atimes B$ is clearly in the product $sigma$ algebra, since the product sigma algebra $mathcal{F}otimes mathcal{G}$ of two measurable spaces is the sigma algebra generated by measurable 'rectangles'. i.e:



$mathcal{F}otimes mathcal{G}= sigmaBig( { Etimes F: Ein mathcal{F}, Fin mathcal{G} } Big)$



Going by this definition, your set $Atimes B$ is Lebesgue measurable in $mathbb{R}^{n+m}$. However going by your criterion, recall first that for $Ain mathcal{L}^n$ and $Bin mathcal{L}^m$, we have:



$lambda_n otimes lambda_m (Atimes B)= lambda_n(A)times lambda_m(B)$



Since $A,B$ are Lebesgue measurable, there exist $U_1,U_2$ open and $V_1,V_2$ closed such that:



$U_1supseteq A supseteq V_1$ , $U_2supseteq B supseteq V_2$ while $lambda_n(U_1setminus V_1)leq frac{1}{2} tilde{epsilon}$ and $lambda_m(U_2setminus V_2)leq frac{1}{2} tilde{epsilon}$



Notice that $U_1times U_2supseteq Atimes B supseteq V_1times V_2$ while $U_1times U_2$ is open and $V_1 times V_2$ is closed. Since you can write:



$U_1times U_2= Big( U_1times V_2 Big) sqcup Big( U_1 times (U_2 setminus V_2) Big)= Big( V_1times V_2 Big) sqcup Big( (U_1setminus V_1)times V_2 Big) sqcup Big( U_1 times (U_2setminus V_2) Big) $



Then:



$lambda_{n+m}Big( (U_1times U_2) setminus (V_1 times V_2) Big)= lambda_{n+m}Big( (U_1setminus V_1)times V_2
Big) + lambda_{n+m}Big( U_1 times (U_2setminus V_2)
Big)= $



$= lambda_n(U_1 setminus V_1)cdot lambda_m(V_2)+ lambda_n(U)cdot lambda_m(U_2 setminus V_2)$



If you assume $lambda_n(U_1), lambda_m(U_2)leq M$, then for $tilde{epsilon}= dfrac{epsilon}{2M}$, you've shown that the difference is of measure less than $epsilon$. Otherwise, there is a standard argument (which I will elaborate if you request) of going from the finite measured case to the $sigma$-finite case.







share|cite|improve this answer












share|cite|improve this answer



share|cite|improve this answer










answered Nov 27 '18 at 10:45









Keen-ameteurKeen-ameteur

1,365316




1,365316












  • $begingroup$
    Thanks for the explanation. If there is a standard way to show it, how would it look like in this case?
    $endgroup$
    – Olsgur
    Nov 27 '18 at 13:56










  • $begingroup$
    I'm unclear on what exactly do you mean by showing in a standard way? The 'standard argument'?
    $endgroup$
    – Keen-ameteur
    Nov 27 '18 at 18:34










  • $begingroup$
    Yes, I mean the standard argument with the $sigma$-finite case.
    $endgroup$
    – Olsgur
    Nov 27 '18 at 23:23


















  • $begingroup$
    Thanks for the explanation. If there is a standard way to show it, how would it look like in this case?
    $endgroup$
    – Olsgur
    Nov 27 '18 at 13:56










  • $begingroup$
    I'm unclear on what exactly do you mean by showing in a standard way? The 'standard argument'?
    $endgroup$
    – Keen-ameteur
    Nov 27 '18 at 18:34










  • $begingroup$
    Yes, I mean the standard argument with the $sigma$-finite case.
    $endgroup$
    – Olsgur
    Nov 27 '18 at 23:23
















$begingroup$
Thanks for the explanation. If there is a standard way to show it, how would it look like in this case?
$endgroup$
– Olsgur
Nov 27 '18 at 13:56




$begingroup$
Thanks for the explanation. If there is a standard way to show it, how would it look like in this case?
$endgroup$
– Olsgur
Nov 27 '18 at 13:56












$begingroup$
I'm unclear on what exactly do you mean by showing in a standard way? The 'standard argument'?
$endgroup$
– Keen-ameteur
Nov 27 '18 at 18:34




$begingroup$
I'm unclear on what exactly do you mean by showing in a standard way? The 'standard argument'?
$endgroup$
– Keen-ameteur
Nov 27 '18 at 18:34












$begingroup$
Yes, I mean the standard argument with the $sigma$-finite case.
$endgroup$
– Olsgur
Nov 27 '18 at 23:23




$begingroup$
Yes, I mean the standard argument with the $sigma$-finite case.
$endgroup$
– Olsgur
Nov 27 '18 at 23:23


















draft saved

draft discarded




















































Thanks for contributing an answer to Mathematics Stack Exchange!


  • Please be sure to answer the question. Provide details and share your research!

But avoid



  • Asking for help, clarification, or responding to other answers.

  • Making statements based on opinion; back them up with references or personal experience.


Use MathJax to format equations. MathJax reference.


To learn more, see our tips on writing great answers.




draft saved


draft discarded














StackExchange.ready(
function () {
StackExchange.openid.initPostLogin('.new-post-login', 'https%3a%2f%2fmath.stackexchange.com%2fquestions%2f3014610%2fcartesian-product-of-lebesgue-measurable-sets-is-lebesgue-measurable%23new-answer', 'question_page');
}
);

Post as a guest















Required, but never shown





















































Required, but never shown














Required, but never shown












Required, but never shown







Required, but never shown

































Required, but never shown














Required, but never shown












Required, but never shown







Required, but never shown







Popular posts from this blog

Biblatex bibliography style without URLs when DOI exists (in Overleaf with Zotero bibliography)

ComboBox Display Member on multiple fields

Is it possible to collect Nectar points via Trainline?